Oraux ENS/X/Mines/Centrale 2023

2

Réponses

  • En quelle année cette question a-t-elle été posée à l'écrit de l'agrégation ?
  • @MrJ Pour le 332, on avait traité un exercice analogue sur le forum :
    Peux-tu redonner de l’exercice analogue ? STP 
  • Guego
    Modifié (December 2023)
    @Chaurien : 2005 (Analyse-probas, question II.A.1.b) : https://agreg.org/data/uploads/sujets/AP/AP05.pdf
    Ça n'est pas formulé exactement de la même manière, mais c'est fondamentalement la même chose.
  • MrJ
    MrJ
    Modifié (December 2023)
    @etanche : Je n'ai pas conservé le lien, mais j'avais trouvé l'exercice joli. Je peux détailler la démonstration.
    Planche 332
    Soient $a$, $b$, $c$ des entiers naturels non nuls. Montrer qu’il existe un $n\in\N$ tel que $\sqrt{n^4 + a n^2 + bn + c} \notin\N$.
    Supposons qu'il existe $(a,b,c)\in\left(\N^\ast\right)^3$ tel que $u_n=\sqrt{n^4 + a n^2 + bn + c} \in\N$ pour tout $n\in\N$.
    1) On vérifie que la suite $v : n \mapsto u_{n+2} - 2 u_{n+1} + u_n$ converge vers $2$.
    2) Comme la suite $(v_n)$ est une suite d'entiers par hypothèse, on en déduit qu'il existe $N\in\N$ tel que 
    $$\forall n\in\N~~\text{avec}~~n> N,\quad u_{n+2}-2 u_{n+1} + u_n = 2.$$
    3) On en déduit qu'il existe $(\alpha,\beta)\in\Q^2$ tel que
    $$\forall n\in\N~~\text{avec}~~n> N,\quad u_n = n^2 + \alpha n + \beta.$$
    4) Finalement, en élevant au carré, on obtient pour tout $n\in\N$ avec $n> N$ que
    $$n^4 + a n^2 + b n + c = n^4 + 2 \alpha n^3 + (\alpha^2 + 2\beta) n^2 + 2 \alpha\beta n + \beta^2,$$
    ce qui est impossible car $b\neq 0$.
  • MrJ
    MrJ
    Modifié (December 2023)
    Planche 129
    Existe-t-il une partie $A$ de $\N$ telle que $\displaystyle{\sum_{n\in A} \dfrac{x^n}{n!} \underset{x\to +\infty}{\sim} e^{\sqrt{x}}}$ ?
    C'est un exercice résolu proposé par @Poirot dans le calendrier de l'avent 2022 sur le forum : Solution de @Namiswan.
  • Merci Guego pour la référence.
  • Chaurien
    Modifié (December 2023)
    Le résultat du 303 est valable pour tout $K$-espace vectoriel de dimension finie, $K$ sous-corps de $\mathbb C$, et pour tout endomorphisme $p$ diagonalisable dont toutes les valeurs propres sont $ \neq \frac12$.
  • MrJ
    MrJ
    Modifié (December 2023)
    Comme je suis sur un ordinateur cette fois-ci, je peux ajouter quelques éléments de solution pour la planche 126 que j'ai mentionné dans un message précédent.
    Planche 126
    Montrer que, pour tout $n\in\N$, il existe $(a_0,\ldots, a_n) \in (\R_+^\ast)^{n+1}$ tel que, pour tout $(\varepsilon_0,\ldots, \varepsilon_n) \in \{−1, 1\}^{n+1}$, le polynôme $\displaystyle{P (X) = \sum_{k=0}^{n} \varepsilon_k a_k X^k}$ est scindé sur $\R$.
    Le résultat se déduit directement du lemme suivant par récurrence sur $n\in\N$.
    Lemme : Si $Q\in\R[X]$ est un polynôme de degré $d\in\N$ scindé à racines simples sur $\R$, alors il existe un réel $\eta_0 > 0$ tel que pour tout $\eta \in \,]{-}\eta_0,\eta_0[$, le polynôme $P_\eta(X) = \eta X^{d+1} + Q(X)$ soit scindé à racines simples sur $\R$.
    Trame de la démonstration. On considère un segment $S$ de sorte que les racines de $Q$ soit dans l'intérieur de $S$. En particulier, la fonction $t\mapsto Q(t)$ change $d$ fois de signe dans $S$. Comme la famille $(P_\eta)$ converge uniformément vers $Q$ sur $S$ lorsque $\eta\to 0$, on en déduit qu'il existe $\eta_0 > 0$ tel que pour tout $\eta \in\, ]{-}\eta_0,\eta_0[$, la fonction $t\mapsto P_\eta(t)$ change également $d$ fois de signe sur $S$. On en déduit par le théorème des valeurs intermédiaires que pour tout $\eta \in\, ]{-}\eta_0,\eta_0[$, le polynôme $P_\eta$ admet au moins $d$ racines dont l'ordre de multiplicité est impair, ce qui implique nécessairement que $P_\eta$ est scindé à racines simples sur $\R$.
  • MrJ
    MrJ
    Modifié (December 2023)
    Pour la planche 52 : c'est une démonstration possible de la décomposition de Frobenius (ou Jordan) pour un endomorphisme nilpotent. On peut par exemple voir la démonstration dans une annexe du livre Les maths en tête : Algèbre de Xavier Gourdon. Il faut légèrement adapter la démonstration, car celle présente dans le livre utilise la notion de base duale, qui est contournée dans l'énoncé de la RMS.
  • MrJ
    MrJ
    Modifié (December 2023)
    Une petite dernière planche ; ensuite je ferai une pause parce que ça tourne au monologue. :p
    Pour la planche 54, si $\varphi : G \to GL_n(\C)$ est un morphisme injectif de groupes où $G=\Z/d_1\Z\times\cdots\times\Z/d_r\Z$, alors tous les éléments de $\varphi(G)$ commutent deux à deux et ils sont annulés par $X^{d_r}-1$. Par un résultat classique, on en déduit qu'il existe $P\in GL_n(\C)$ tel que $P \varphi(G) P^{-1}$ est un sous-groupe de $\{\textrm{diag}(\lambda_1,\ldots,\lambda_n)\mid \lambda_1,\ldots,\lambda_n \in\mathbb{U}_{d_r}\}$. Ce dernier groupe étant isomorphe à $(\Z/d_r\Z)^{n}$, on en déduit que l'on peut remplacer $GL_n(\C)$ par $(\Z/d_r\Z)^{n}$ dans la question initiale.
    Finalement, en remarquant que l'équation $d_1 x = 0$ admet $d_1^r$ solutions dans $G$ et $d_1^n$ solutions dans $(\Z/d_r\Z)^{n}$, on conclut que $n\geq r$ nécessairement. Réciproquement, on vérifie que $n=r$ est l'entier minimal cherché.
  • Dagothur
    Modifié (December 2023)
    @MrJ J'ai fait exactement le même raisonnement en voyant cet énoncé (la 54)
  • Chaurien
    Modifié (December 2023)
    Je suis étonné par l'exo double étoile 277 :
    ..............................................................................................................................................................................................................................
    277 ⋆⋆  a) Montrer que l’équation $a^2-2b^2=1$ admet une infinité de solutions $(a, b) \in \mathbb N^2$. Déterminer l’ensemble des solutions. 
    b) Que dire de l’ensemble des solutions de $a^2-2b^2=-1$ ?
    ..............................................................................................................................................................................................................................
    Faut-il feindre de redécouvrir la théorie classique de l'équation de Fermat-« Pell » ? Pourquoi cette question initiale sur l'équation $a^2-2b^2=1$ augmentée d'une seconde question sur $a^2-2b^2=-1$, alors que chacun sait que cette équation se traite d’emblée comme $a^2-2b^2=\pm1$ ? 
    Exo de Gribouille...
  • Guego
    Modifié (December 2023)
    Par le binôme de Newton, pour tout $n\in \N$, il existe $(a_n,b_n)\in \N^2$ tel que $(1+\sqrt{2})^n = a_n + b_n\sqrt{2}$ et $(1-\sqrt{2})^n = a_n - b_n\sqrt{2}$. On en déduit $a_n^2 - 2b_n^2 = (-1)^n$ pour tout $n\in \N$.
  • Je sais démontrer qu'il n’y a pas d’autres solutions (que celles données pas Guego) en déterminant les unités de l’anneau $\Z[\sqrt{2}]$, mais je me demande si on peut facilement le prouver sans passer par cet anneau ?
  • etanche
    Modifié (December 2023)
    Pour le 277 historique et théorie sur Pell-Fermat 
    https://webusers.imj-prg.fr/~leonardo.zapponi/leo_divers/4M033/Chapitre1.pdf
    on y trouve la célèbre équation du problème des bœufs d’Hélios $x^2-410286423278424y^2=1$.
  • Ben oui, justement, cette équation de Fermat-« Pell » est archi-classique. Dans les années 1970-80, on posait ça au bac C avec des matrices $2 \times 2$. On peut exposer la solution sans référence à la structure d'anneau, c'est ce qui se faisait dans les traités de théorie des nombres de la première moitié du XXème siècle (Sierpiński, LeVeque, Niven-Zuckerman, etc.), mais la meilleure méthode à mon avis c'est celle que rappelle MrJ. Je répète que je me demande ce que fait là cet exercice, surtout posé ainsi, aussi mal que possible, d'abord $a^2-2b^2=1$ puis $a^2-2b^2=-1$.
  • Guego
    Modifié (December 2023)
    @MrJ : on doit pouvoir montrer par récurrence sur $n$ que toute solution $(a,b)$ où $a\leqslant n$ est de la forme souhaitée.
    Pour l'hérédité, si $(a,b)\in \N^2$ est une solution de $a^2-2b^2 = 1$, alors $(c,d):=(3a-4b, -2a+3b)$ en est une autre plus petite (vérification laissée au lecteur). Par hypothèse de récurrence, il existe $k$ tel que $c+d\sqrt{2} = (1+\sqrt{2})^k$. Or, $a+b\sqrt{2} =(c+d\sqrt{2})(1+\sqrt{2})^2$ (c'est en partant de ça que j'ai construit $c$ et $d$ au brouillon au préalable), d'où $a+b\sqrt{2} = (1+\sqrt{2})^{k+2}$.
  • JLapin
    Modifié (December 2023)
    Je répète que je me demande ce que fait là cet exercice, surtout posé ainsi, aussi mal que possible

    Il y a un bon millier d'exercices dans chaque numéro de la RMS, issus de retour effectués par des élèves en plein été pendant leurs oraux. Ce n'est peut-être pas indispensable de pointer publiquement du doigt de façon vindicative tel ou tel énoncé mal ficelé et/ou peu original parmi cette longue liste. Un peu de charité fraternelle serait appréciable, non ?

  • etanche
    Modifié (December 2023)
    98 je l'avais déjà vu dans un problème d'écrit de concours. 
    Les élèves qui ont eu cet exercice à l'oral ont dû transpirer.
  • MrJ
    MrJ
    Modifié (December 2023)
    @Guego : Merci ! Même si la rédaction est totalement différente, l'idée est assez proche de la solution passant par l'anneau $\Z[\sqrt{2}]$ consistant à trouver l'unité fondamentale dans $]1,+\infty[$.
  • etanche
    Modifié (December 2023)
    Pour le 27 est-ce que les scalaires $\lambda, \mu$ sont plutôt dans $\R$ au lieu de $\R^{*}$ ?
  • etanche
    Modifié (December 2023)
    424 c’est Erdös-Suryani 
    https://fr.wikipedia.org/wiki/Théorème_d'Erdős-Suranyi

    100 la célèbre série de G.Vacca new series Eulerian constant Quart.J.Pure Appl. Math. 41 (1910) 363-364
  • bisam
    Modifié (December 2023)
    Pour le 59, on remarque tout d'abord la coquille qui a fait écrire une somme allant jusqu'à $n$ au lieu de $m$. $\newcommand{\ps}[2]{\left\langle #1, #2 \right\rangle}$
    Notons $U=\text{vect}(u_1,\dots,u_m)$ et $q$ l'endomorphisme de $E$ qui à tout vecteur $x$ associe $\displaystyle q(x)=\sum_{i=1}^m \ps{x}{u_i}v_i$.
    Alors on vérifie que $q$ est un projecteur et que pour tout $x$ dans $E$ et tout $i$ dans $\{1,\dots, m\}$, $\ps{q(x)}{u_i}=\ps{x}{u_i}$ donc $x-q(x)\in U^{\bot}$. Ensuite, puisque le projecteur $p$ est orthogonal, il est autoadjoint et on calcule : \[\begin{align}\sum_{i=1}^m \ps{u_i}{x} \ps{x}{p(v_i)} &= \sum_{i=1}^m \ps{u_i}{x} \ps{p(x)}{v_i} \\ &=\ps{p(x)}{\sum_{i=1}^m \ps{u_i}{x} v_i} \\ &=\ps{p(x)}{q(x)} \\ &= \ps{p(x)}{p(x)} + \underbrace{\ps{p(x)}{x-q(x)}}_{=0}  + \underbrace{\ps{p(x)}{p(x)-x}}_{=0} \\ &= \|p(x)\|^2 \end{align}\]
  • La différence entre le 343 et le 414 me paraît subtile.
    Il y a sans doute une question de densité qui se cache.
  • 91 $v_n$ tend vers $\varphi$, $w_n$ tend vers $\ln(\varphi)$ où $\varphi$ le nombre d’or.
  • etanche
    Modifié (December 2023)
    Comment feriez-vous le 133 ?
    Merci.
  • Guego
    Modifié (December 2023)
    Je ne suis pas allé au bout, mais quelques remarques.
    Déjà, l'intégrale de $0$ à $\dfrac{\pi}{2}$ est égale à l'intégrale de $\dfrac{\pi}{2}$ à $\pi$, donc $f(x) = 2 \displaystyle \int_0^{\pi/2} \dfrac{1}{\sqrt{\cos^2(t) + e^{-4x}\sin^2(t)}} dt$. Ensuite, en mettant en facteur $e^{2x}$ sous la racine :  \[ f(x) = 2e^{-x} \displaystyle \int_0^{\pi/2} \dfrac{1}{\sqrt{\cos^2(t) + e^{-4x}\sin^2(t)}} dt  = 2e^{-x} \displaystyle \int_0^{\pi/2} \dfrac{1}{\sqrt{\sin^2(t) + e^{-4x}\cos^2(t)}} dt \] En posant $g(x) = \displaystyle \int_0^{\pi/2} \dfrac{1}{\sqrt{\sin^2(t) + e^{-4x}\cos^2(t)}} dt$, la fonction $g$ est continue sur $[0;+\infty[$. Si on trouve un équivalent du type $g(x) \underset{+\infty}{\sim} Ax$, avec $A$ constante, c'est gagné.
  • Guego
    Modifié (December 2023)
    J'avance : le changement de variable $u = \sin^2(t)$ donne $g(x) = \dfrac{1}{2} \displaystyle \int_0^1 \dfrac{1}{\sqrt{u(1-u)(u+e^{-4x}(1-u))}}du$. Or, $ \dfrac{1}{\sqrt{u(1-u)(u+e^{-4x}(1-u))}} - \dfrac{1}{\sqrt{u(u+e^{-4x})}}$ tend, quand $x$ tend vers $+\infty$ vers $ \dfrac{1-\sqrt{1-u}}{u\sqrt{1-u}}$, qui est intégrable sur $[0;1]$.
    Donc, par convergence dominée (à justifier), on a $g(x) - \dfrac{1}{2} \displaystyle \int_0^1 \dfrac{1}{\sqrt{u(u+e^{-4x})}} du = C+o(1)$, où $C = \displaystyle  \dfrac{1}{2} \int_0^1 \dfrac{1-\sqrt{1-u}}{u\sqrt{1-u}}du$.
    Reste à étudier $\displaystyle \int_0^1 \dfrac{1}{\sqrt{u(u+e^{-4x})}} du$, mais ça se calcule explicitement (avec un $argsh$). On trouve, après calculs : $4x + \ln\Big(2\sqrt{1 + e^{-4x}} + 2 + e^{-4x}\Big) = 4x + 2\ln(2) + o(1)$
    Conclusion : $f(x)e^x = 4x + B + o(1)$ quand $x$ tend vers $+\infty$, avec $B$ une constante. D'où le résultat.
    Édit : la constante $C$ se calcule facilement (poser $v=\sqrt{1-u}$), on trouve $C = 2\ln(2)$. D'où $f(x)e^x = 4x+4\ln(2)+o(1)$.
  • etanche
    Modifié (December 2023)
    103 c’est le théorème de Flett me semble-t-il
    A Mean Value Theorem, T. M. Flett, The Mathematical Gazette, Vol. 42, No. 339. (Feb., 1958), pp. 38-39.
    la preuve de son théorème par M.T.Flett https://people.math.sc.edu/girardi/m555/current/hw/MVT-Flett.pdf
  • morlock84
    Modifié (December 2023)
    Je me lance pour le 168 

    On note $f$ la fonction génératrice d'une telle variable aléatoire $X$. Les hypothèses nous donnent par un calcul simple $f(1) = f'(1) = f''(1) = f'''(1) = 1$. Il s'agit donc de donner un minorant optimal pour $f(0)$. Par théorème de Taylor-Lagrange reste intégral on a $f(0)-f(1)+f'(1)-\frac{f''(1)}{2} = f(0) - \frac{1}{2} = -\int_0^1 \frac{t^2 f'''(t)}{2} dt \geq -\int_0^1 \frac{t^2}{2} = -\frac{1}{6}$, car $f'''$ est croissante sur $\left[0,1\right]$. Ainsi $f(0) \geq \frac{1}{3}$. On a égalité si et seulement si $f'''(t) = 1$ sur $\left[0,1\right]$, i.e si $f$ est sur cet intervalle un polynôme de degré $3$ de coefficient dominant $\frac{1}{6}$. Les conditions sur les dérivées en 1 nous donnent les coefficients de ce polynôme : on en déduit que la variable aléatoire définie par $P(X=0) = \frac{1}{3}$, $P(X=1) = \frac{1}{2}$ et $P(X=3) = \frac{1}{6}$ vérifie les hypothèses demandées. $\frac{1}{3}$ est donc optimal.
  • Bonjour, c'est en raisonnant modulo $8$ que vous faites 519? Cet exercice m'intéresse mais pour le moment, je n'aboutis pas à grand chose, je dois m'y prendre mal... 
    Lorsque notre cher Gebrane, le 😄 farceur, intervient dans une question d'algèbre, c'est une véritable joie pour les lecteurs.


  • Parku
    Modifié (December 2023)
    Bonjour NicoLeProf, j'ai distingué selon la parité de $m$. 
    Soit $(m,n)$ un couple solution.
    • Si $m$ est pair, alors $3^m = \big(3^{m/2}\big)^2$ est un carré, donc $8 = 3^m - n^2$ est la différence de deux carrés. On voit facilement que la seule différence entre deux carrés qui vaut $8$ est atteinte pour $3^2 - 1^2$ (les sauts entre les carrés deviennent ensuite trop grands). On a donc $m = 2$ et $n= 1$.
    • Si $m$ est impair, alors $3^m = 3 \mod 8$. On constate que les carrés ne valent jamais $3$ modulo $8$, donc il n'y a pas de solution possible.
  • NicoLeProf
    Modifié (December 2023)
    "les sauts entre les carrés deviennent ensuite trop grands" : désolé, pas convaincu par cette phrase. 
    Si $m=4$ et $n=9$, on a : $3^m-n^2=3^4-9^2=0$ donc je ne sais pas ce que cette phrase ci-dessus signifie.
    Comment avez-vous raisonné @etanche, @Chaurien? (Une indication du modulo utilisé si vous voulez me laisser chercher ! :D)
    Lorsque notre cher Gebrane, le 😄 farceur, intervient dans une question d'algèbre, c'est une véritable joie pour les lecteurs.


  • etanche
    Modifié (December 2023)
    @ nicoleprof  regarde la solution de koshrotash sans modulo
    https://math.stackexchange.com/questions/3768473/prove-that-x2-8-3y-has-only-one-solution-x-1-y-2-where-x-y-in-mat?noredirect=1
    Solution avec modulo par ici https://artofproblemsolving.com/community/c6h1717143p11094629
    Le 519 était un exercice de Thailand IPST Math camp problem
  • Parku
    Modifié (December 2023)
    @NicoLeProf C'est pourtant très élémentaire. La phrase avant la parenthèse signifie que les seuls entiers naturels $a,b$ tels que $b^2 - a^2 = 8$ sont $b =3$ et $a = 1$. La phrase entre parenthèse "les sauts entre les carrés deviennent ensuite trop grands" est une explication condensée dont voici les détails.
    Soit $a,b \in \mathbb{N}$ tels que $a \leq b$. Alors \[b^2 - a^2 = 2(b-a)a+(b-a)^2.\]
    Si $b^2 - a^2 = 8$, alors en raisonnant modulo $2$, $b-a$ doit être pair, $b-a$ est évidemment non nul et strictement inférieur à $3$ (sans quoi $b^2 - a^2 \geq 9$), donc $b-a = 2$, puis $8 = 4a+4$ donc $a = 1$, d'où finalement $a=1$ et $b=3$.
    On vérifie que, réciproquement, $3^2 -1^2 =8$.
  • NicoLeProf
    Modifié (December 2023)
    Ok merci beaucoup pour les détails Parku, j'ai compris !
    Merci aussi etanche pour les autres solutions !
    Lorsque notre cher Gebrane, le 😄 farceur, intervient dans une question d'algèbre, c'est une véritable joie pour les lecteurs.


  • Sinon, bêtement : $b^2-a^2 = (b+a)(b-a)$. Ainsi, si $b^2-a^2 = 8$, avec $0\leqslant a  \leqslant b$, alors $(b+a,b-a) = (8,1)$ ou $(b+a,b-a) = (4,2)$. Le premier cas donne des $a$ et $b$ non entiers, et le deuxième cas donne $a=1$ et $b=3$.
  • etanche
    Modifié (1 Jan)
    Comment vous faites pour le 1? 
    C’est un exercice original pour le 1 Janvier 2024, d’ailleurs c’est le numéros 1 du fichier de la RMS.
    C’est rare de voir un exercice de concours avec la fonction $\lceil a \rceil$
    $\lceil \frac {m}{n} \rceil $ fait penser aux principes des tiroirs
    $xR_fy <=> f(x)=f(y)$ est une relation d’équivalence sur $S$
    On a une bijection de $S/R_f$ sur $f(S)$
    $X=\{ (x,y)\in S^2 , f(x)=f(y) \}$ est appelé le noyau de l’application $f$
    On remarque aussi $X=\{ f^{-1}(y), y\in Im(f) \}$
    $\vert X \vert = \sum_{y \in Im(f)} card(f^{-1}(y))$ 

    Une expression similaire à $\frac{(\vert S \vert )^2}{\vert T \vert}$ se retrouve dans le nombre de classe de conjugaisons 
    $k(G)\geq \frac{(\vert H \vert)^2 }{\vert G \vert }$ 
    https://math.stackexchange.com/questions/2607252/the-number-of-conjugacy-classes-of-a-finite-group?noredirect=1
  • @ Chaurien le 2 c’est Putnam 2006 problème B2  https://kskedlaya.org/putnam-archive/2006.pdf 
  • Ben314159
    Modifié (1 Jan)
    Salut,
    Pour le 1), j'aurais commencé par simplifier les notations en prenant $m\!=\!|T|$ et, si $T=\{y_1,y_2,\dots,y_m\}$ en posant, $\lambda_i\!=\!|f^{-1}(y_i)|$ pour tout $i\!\in\!\{1..m\}$.  Avec ces notations, ce que tu as, c'est $\displaystyle|S|\!=\!\sum_{i=1}^m\!\lambda_i$ et $\displaystyle|X|\!=\!\sum_{i=1}^m\!\lambda_i^2$ et absolument rien d'autre.
    Montrer que $|X|\!\geqslant\!\dfrac{|S|^2}{|T|}$, ça signifie montrer que $\displaystyle m\sum_{i=1}^m\lambda_i^2\geqslant\big(\sum_{i=1}^m\lambda_i)^2$ et c'est l'inégalité de Cauchy-Schwarz.
    Pour l'autre inégalité il suffit de dire au moins un des $\lambda_i$ est $\geqslant\!\lceil \mu\rceil$ où $\mu\!=\!\dfrac{|S|}{|T|}$ est la moyenne des $\lambda_i$,  et que, pour les autres, on a $\lambda_i^2\!\geqslant\!\lambda_i$.
    (on pourrait faire mieux comme minoration en montrant que, pour rendre $\displaystyle \sum_{i=1}^m\!\lambda_i^2$ la plus petite possible pour $\displaystyle |S|=\sum_{i=1}^m\!\lambda_i$ fixés, il faut prendre tout les $\lambda_i$ égaux soit à $\lceil \mu\rceil$, soit à $\lceil \mu\rceil\!-\!1$)
  • @ ben314519 pour la deuxième inégalité comment arrive-t-on à $\geq \lceil \mu \rceil^2 + \vert S \vert - \lceil \mu \rceil $
    peux-tu détailler davantage merci
  • Ben314159
    Modifié (2 Jan)
    Les $\lambda_i$, il y en a au moins un qui est plus grand que la moyenne  $\displaystyle \mu\!=\!\dfrac{1}{m}\sum_{i=1}^m\lambda_i=\dfrac{|S|}{|T|}$ et, comme c'est un entier, il est en fait $\geqslant\lceil\mu\rceil$. 
    Considérons pour simplifier la rédaction que c'est $\lambda_1$.  On a alors 
    $\displaystyle \sum_{i=1}^m\lambda_i^2=\lambda_1^2+\sum_{i=2}^m\lambda_i^2\geqslant \lambda_1^2+\sum_{i=2}^m\lambda_i=\lambda_1^2+\sum_{i=1}^m\lambda_i-\lambda_1\geqslant\lceil\mu\rceil^2+|S|-\lceil\mu\rceil\ \ $ vu que $\ n\mapsto n^2\!-\!n\ $ est croissante sur $\mathbb N$.
  • etanche
    Modifié (2 Jan)
    @ Ben 314519 de jolies idées dans ta solution 

    Encore un détail qui m’échappe comment on établit que $\vert X \vert = \sum_{i=1}^{m} \lambda_{i}^2$ 
    pourquoi il y a une puissance de 2?merci
  • Ben314159
    Modifié (2 Jan)
    $\displaystyle X=\bigcup_{i=1}^{m}\big\{(x,x')\!\in\!S^2\mid f(x)\!=\!f(x')\!=\!y_i\big\}=\bigcup_{i=1}^{m}\big(f^{-1}(y_i)\big)^2,\ $
    où les $\ \big(f^{-1}(y_i)\big)^2\!\subset\!S^2\ $ sont évidement disjoints (et de cardinal $\lambda_i^2$).
  • etanche
    Modifié (5 Jan)
    Finalement pour résoudre cet exercice nécessite des connaissances de 1ère année sur la théorie des ensembles,
    cardinaux, inégalité Cauchy-Schwarz. C’est un exercice difficile, et peut-être même déstabilisant à l’oral.
    Mais un joli exercice pour un oral ENS ou l’X.

  • morlock84
    Modifié (4 Jan)
    C'est moi où le 105 est ultra simple, en plus d'avoir des hypothèses bien trop larges ? ça me parait bizarre, peut-être que je vais dire une énormité. Si $f^2 = 0$, on a $f =0$. Si maintenant $f^2 \neq 0$, on pose $I = \left]b,+\infty\right[$ avec $b$ la plus grande racine réelle de $f^2$, ou $I = \mathbb{R}$ si $f^2$ n'en a pas. Sur $I$ on peut écrire $f = \frac{f^3}{f^2}$, donc $f$ est une fonction rationnelle sur cet intervalle. On note $P$ et $Q$ les polynomes associés à $f^3$ et $f^2$ respectivement : on a $P^2 = Q^2 \times Q$ (égalité des fonctions polynomiales associées sur une infinité de réels), donc en particulier $Q^2$ divise $P^2$. Donc $Q$ divise $P$, en passant par la décomposition en irréductibles de $\mathbb{C}\left[X\right]$ par exemple. Donc $f$ est polynomiale. 

    Edit : c'est vrai que le raisonnement ne marche seulement que sur tous les intervalles dont les bornes sont deux racines consécutives de $f^2$ (ou de $f$), et qu'il faut ensuite raccorder, mais cela ne semble pas poser de problèmes non plus vu que $f$ est nécessairement continue en ces points ?
  • etanche
    Modifié (5 Jan)
    @ morlock 84 parfois aux ENS y a des exercices faciles niveau sup, regarde le 102 par exemple, c’est la fonction Thomae discontinue sur les rationnels, et continue sur les irrationnels.
    https://en.wikipedia.org/wiki/Thomae%27s_function 
    Une vidéo sur la fonction Thomae https://www.youtube.com/watch?v=4lcYtmKNqmE

    Une étude de la continuité de la fonction Thomae 
    https://sites.math.washington.edu/~morrow/334_15/thomae.pdf

    Prolongement du 102 
    f est-elle périodique ? Étudier la derivabilité, les extrémums locaux. f est-elle Riemann integrable ?
    Sur la dimension du graphe de la fonction Thomae https://arxiv.org/pdf/2007.08407.pdf#page3
  • Renart
    Modifié (6 Jan)
    Morlock : On peut raccorder deux polynômes de façon continue sans que le résultat obtenu soit un polynôme, par exemple $x \mapsto |x|$. Il faut donc faire un petit peu attention tout de même.
    Etanche : Les prolongements proposés sont intéressants, je crois que je ne m'étais jamais posé certaines de ces questions (extrema et dérivabilité). Après quelques gribouillis à la va vite sur une feuille de papier. Périodique (si $f(0)=1$) de période $1$, dérivable nulle part, chaque rationnel est un maximum local, chaque irrationnel est un minimum global, la fonction est Riemann-intégrable sur tout segment. À noter que comme l'ensemble des discontinuités de $f$ est négligeable on obtient immédiatement qu'elle est intégrable au sens de Riemann mais c'est intéressant de le montrer "à la main" en revenant à l'encadrement par des fonctions étagées.
  • morlock84
    Modifié (6 Jan)
    @Renart Certes, mais si je ne me trompe pas le polynôme est le même sur chacun des intervalles, mon raisonnement étant le même si on change de $I$. Il reste juste à dire que $f$ est continue aux éventuels points où elle s'annule, ce qui découle du fait que $f^2$ vérifie cette même propriété.
Connectez-vous ou Inscrivez-vous pour répondre.